Finding asymptotes of parametric equations

Click For Summary
To find the asymptotes of parametric equations, it's important to analyze the behavior of x and y as x approaches large values or zero. The first problem's solution is correct, noting that as x approaches 0, y equals pi/4, indicating the y-intercept. The second problem does not have any asymptotes, as both x and y approach infinity in opposite directions. For intercepts, set x to 0 to find y-intercepts and set y to 0 to find x-intercepts by solving for t in each case. Understanding these concepts is crucial for accurately determining asymptotic behavior in parametric equations.
fluppocinonys
Messages
19
Reaction score
1
Find the asymptotes of the parametric equations
29blged.gif


The graph looks like this:
qmOcE.jpg





My attempt:
4clfb.gif

Is my presentation correct? Do I also need to consider as x approaches 0 ?



The reasons I ask is because I couldn't find the asymptotes of
G0dMb.gif

if I use the above presentation...
Please guide me...
Thanks.
 

Attachments

  • 29blged.gif
    29blged.gif
    1.1 KB · Views: 626
Physics news on Phys.org
The first problem looks fine, although you could have done it without solving for t in the first equation. Just look at what happens to x and y as x gets large or approaches zero. If x = 0, y = pi/4. All this is, is the y-intercept.

For the second problem, there aren't any asymptotes. As x --> infinity, x --> -infinity and y --> +infinity. As x --> -infinity, x --> -infinity and y --> -infinity. For intercepts, set x = 0 and solve for t, then substitute it into the other equation. That will give you the y-intercept(s). Set y = 0 and solve for t, then substitute it into the other equation, to get the x-intercepts(s).
 
Question: A clock's minute hand has length 4 and its hour hand has length 3. What is the distance between the tips at the moment when it is increasing most rapidly?(Putnam Exam Question) Answer: Making assumption that both the hands moves at constant angular velocities, the answer is ## \sqrt{7} .## But don't you think this assumption is somewhat doubtful and wrong?

Similar threads

  • · Replies 7 ·
Replies
7
Views
2K
  • · Replies 5 ·
Replies
5
Views
2K
  • · Replies 3 ·
Replies
3
Views
3K
  • · Replies 3 ·
Replies
3
Views
3K
  • · Replies 4 ·
Replies
4
Views
3K
  • · Replies 2 ·
Replies
2
Views
2K
  • · Replies 11 ·
Replies
11
Views
3K
  • · Replies 11 ·
Replies
11
Views
3K
  • · Replies 10 ·
Replies
10
Views
2K
  • · Replies 6 ·
Replies
6
Views
2K